This is a Resolve, Reconcile, and Explain Question. We know this because of the question stem: “Most helps to resolve the apparent discrepancy?”
RRE questions will require an explanation of a conflicting set of facts. Our correct answer choice, when plugged back into the stimulus, will resolve the discrepancy by explaining how the two sides of the apparent conflicting issues actually make sense together. The correct answer will use both sides, though not necessarily explicitly, to explain the conflict. Often, the test will entice you to make naive assumptions about the conflict - don’t fall for it! Your approach should fall under the “this seems wrong because of XYZ, but I can think of a few reasons it could work.”
The first sentence reports on what the city’s CDC says about rabid raccoons: it’s more serious now than it was two years ago because of a percentage increase in raccoons with rabies. However, the second sentence gives us an interesting statistic: the number of confirmed cases has gone down from two years ago to this year.
This plays off a very important distinction, which often comes up in flaw-type questions. This stimulus is comparing percentages to numbers. There is a whole host of reasons based on this alone that could explain this discrepancy. Other reasons could be that the data doesn’t accurately reflect the actual confirmed cases of rabid raccoons.
Answer Choice (A) Other animals? Does this explain the discrepancy between the percent and numbers? Nope. This is out.
Correct Answer Choice (B) With a significant portion of raccoons succumbing to rabies, this could explain the apparent discrepancy between numbers and percentages. Let's say two years ago there were 130 rabid raccoons and <25% had rabies, so 32 raccoons. Now, since there is a substantial decrease, it's possible that the raccoon population is 34, and 18 of them have rabies. 18 is still over 50% of 34 - this would explain the numbers! If you’re thinking “There are fewer of them in general, so how is this a serious problem??” However, a greater proportion of raccoons having rabies is still a pretty serious problem.
Answer Choice (C) This would be correct if we are assuming that the cases of rabies counted a few years ago were confused with distemper, and so the actual number of confirmed cases two years ago is a lot lower. However, that’s a pretty big assumption, one we cannot make. Without this assumption, the answer isn’t good.
Answer Choice (D) Outside information is okay, but saying that these animals are nocturnal does not help explain the conflict here.
Answer Choice (E) The adds more information to the facts but does not help resolve. This is out.
This is a Resolve, Reconcile, and Explain Question. We know this because of the question stem: “...most helps to resolve the apparent discrepancy?”
RRE questions will require an explanation of a conflicting set of facts. Our correct answer choice, when plugged back into the stimulus, will resolve the discrepancy by explaining how two sides of the apparent conflicting issues actually make sense together. The correct answer will use both sides, though not necessarily explicitly, to explain the conflict. Often, the test will entice you to make naive assumptions about the conflict - don’t fall for it! Your approach should fall under the “this seems wrong because of xyz, but I can think of a few reasons it could work.”
Our first sentence presents us with what happens when you consume excess calories: you gain weight. This makes sense. Our next sentence says that alcoholic beverages have tons of calories. This (sadly) tracks.
Before we read on, with these two ideas in mind, what can we infer? That excess intake of alcoholic drinks could cause someone to gain weight. With this question, it’s possible to anticipate where the argument could go; they’re probably going to say something about how, contrary to these facts, people who drink a lot of alcohol don’t gain weight.
What does our next sentence say? Exactly this: people who drink 2-3 drinks, exceeding their intake, do not generally gain weight. The grammar here is a little complicated. If helpful, focus on the subject and then the predicate and then expand your scope from there. The subject is “people.” What kind of people? And so on.
Back to the argument. Can we think of reasons why people who drink 2-3 beverages a day do not gain weight? Perhaps these people exercise a lot. Perhaps these people have higher metabolisms. These are just two reasons - they are many, many more.
Okay - let’s go to the answer choices.
Answer Choice (A) This may seem attractive, but it’s absolutely wrong. We’re directly rejecting the facts we’re given. The people who drink these excess calories do exceed their caloric intake - it’s in the last sentence. This is out.
Correct Answer Choice (B) It’s giving an overlooked possibility for how these people who drink 2-3 drinks are able to not gain weight. The excess calories are dissipated through heat, which means they’re not converted and stored as fat.
Answer Choice (C) This answer choice focuses on people who do not drink but who eat high-calorie foods and still do not gain weight. How does this help resolve the fact that people who drink excess calories don’t gain weight?
Answer Choice (D) This is a restatement of the last sentence! It doesn’t add or explain anything.
Answer Choice (E) This is completely irrelevant to our issue: people who don’t eat excess calories don’t lose weight. Don’t lose weight? This doesn’t even interact with either of our sides.
This is a most strongly supported question, as the question stem asks: The statements above, if true, most strongly support which one of the following conclusions?
Our stimulus begins with a description of rheumatoid arthritis, specifically the mechanism by which it occurs. Immune system misfunction leads to attacks on joint cells which causes the release of a hormone which in turn causes pain and swelling. So far all we have is a causal chain; immune misfunction > join cells attacked > hormone released > pain and swelling. The next sentence gives us a bit more information on this hormone; it is normally only triggered by injury and infection. Since we know the hormone causes pain and swelling, what we should infer from this is that injury or infection can also lead to the same pain and swelling as rheumatoid arthritis. Finally, we are told about a new medication that includes a protein that blocks the pain and swelling hormone. Let’s see what the correct answer infers from this:
Answer Choice A We are only told that the medication “contains” a hormone-inhibiting protein for a hormone that specifically causes pain and swelling; nothing in the stimulus mentions or suggests that it also repairs cells. This answer is consistent with the stimulus, but completely unsupported.
Answer Choice B This is consistent with what we know but not something any of the information we have been given would support. We are told nothing about how the medications benefits compares to any possible harmful side effects.
Correct Answer Choice (C) This is where the inference we made from the second sentence comes in handy. We know that the hormone can be caused by injury and we know that it causes pain; if the medication stops this pain that would result from an injury, then we have reason to believe that at least some of the pain that results from a join injury wouldn’t occur because of the medication. C is the only answer that has any significant support in the stimulus.
Answer Choice (D) We have been told nothing about the mechanisms by which other immune system disorders cause pain. We are not entitled to make the assumption that they are similar enough to rheumatoid arthritis that the medication could be adapted, and therefore this answer has little support.
Answer Choice (E) Similar to D, we have been given no information about other joint diseases. Maybe they do all involve the same hormone.
This is a most strongly supported question, as the question stem asks: If the statements above are true, which one of the following is most strongly supported by them?
Our stimulus begins by telling us that in the last quarter century the introduction of labor-saving tech has greatly reduced the average amount of time required for a worker to produce a corresponding output. This change has the potential to reduce the number of hours workers spend working every work, and increasing the amount of time they have to themselves. Who doesn’t like the sound of that! Unfortunately, it seems the average amount of leisure time has increased at only half the rate that output has grown. What gives! Since this is a MSS question, we should treat all this information as a series of premises. The correct answer will be the conclusion which can follow from these premises with the least assumptions required. Let’s see what are options are like:
Answer Choice (A) This conclusion requires a lot of assumptions. More specifically, we have been told nothing about what workers are spending their money on. Even if they have more leisure time total, what if leisure activities have gotten much cheaper?
Answer Choice (B) We have only been told about what these technologies have done to worker output and leisure time, we know nothing about the amount of jobs they have created or replaced.
Answer Choice (C) Again, this just isn’t something we know much about. For this to follow from the information in the stimulus, we have to make a lot of assumptions.
Answer Choice (D) We’ve been told nothing about what was anticipated compared to the increase that actually occurred.
Correct Answer Choice (E) If the average hourly output has increased, and at double the rate at which average leisure time has increased, then it is highly likely that the average weekly output is higher than it was before the introduction of labor-saving technology. The weekly output is determined by the amount of hours worked in a week and the hourly output, and we know that hours worked per week has declined (leisure time) less than hourly output has increased.
We know this is a sufficient assumption question because of the question stem: “which one… if assumed… would allow [the author] to draw her properly to draw her conclusion…” Interestingly, the conclusion is given to you in the stem. This should help in our formal analysis to figure out what the conclusion is.
Sufficient assumption questions tend to be very formal. We’re looking for a rule that would validate the conclusion, specifically by bridging the premise and conclusion through the rule. Not only are we extrapolating the rule from our argument, but we’re also using that rule to render the argument “valid.” The way to prephrase our answer choice is by tying our premises and conclusion together into a rule: “If [premise] → then [conclusion].”
Our first sentence tells us that an antitheft alarm may stop an attempted theft at night on a crowded street. I’m imagining an incredibly loud siren going off in a middle of a downtown area... I think it’s possible that theft could be prevented with a loud alarm like that.
The next sentence gives an alternative cause for the alarm going off: instead of a thief, it could be a branch or another form of contact.
The third sentence puts aside the causes and says in any of the situations in which the alarm goes off at night, it’s disturbing the sleep of people in the neighborhood. Makes sense - a blaring alarm would definitely disturb my sleep!
Our last sentence, the conclusion which we read in our stem, is: out of consideration for our neighbors, we should deactivate the alarms when parking in crowded city neighborhoods at night.
Why should they?? Protecting a car is probably more important and valuable to its owner than someone else’s sleep, and there doesn’t seem to be a moral code that says people should deactivate their cars if it bothers people’s sleep. On the chance that the car is getting stolen, an alarm would be the best thing to help avoid losing the car! This prescriptive conclusion is assuming that others’ sleep is more important than protecting someone’s car.
What we need is a rule that bridges our premises to our conclusion and validates this prescription. Our premise here is that if the alarm goes off, then people’s sleep is disturbed, and we should be considerate of this. Our conclusion is that owners of these cars should deactivate their alarms. Put them together to make our rule! If people’s sleep is getting disturbed and we should be considerate of this → owners should deactivate car alarms at night.
Answer Choice (A) This would weaken the argument! This is putting the protection of peoples’ cars over peoples’ sleep.
Answer Choice (B) In most cases? What about some cases in which it’s actually a theft? And besides this, the answer choice still does not justify why we should deactivate our alarms. Just because it is a false alarm, it does not validate the argument.
Correct Answer Choice (C) While it’s not a perfect paraphrase of our rule, it gets to the idea that sleep is more important than protecting cars.
Answer Choice (D) Remember, our conclusion is prescriptive. How does saying “people who have alarms are inconsiderate” help our argument? Does this mean they should deactivate their alarms? No – this is out.
Answer Choice (E) We don’t really care about what happens to the alarms during the day, we’re concerned with why people should turn alarms off at night. This is supplemental information that does nothing for our stimulus.
This is a Sufficient Assumption (SA) question and we know this because of the question stem: “conclusion is properly drawn from the premises given if which one of the following is true...”
Sufficient assumption questions tend to be very formal. We’re looking for a rule that would validate the conclusion, specifically by bridging the premise and conclusion through the rule. Not only are we extrapolating the rule from our argument, but we’re also using that rule to render the argument “valid.” The way to prephrase our answer choice is by tying our premises and conclusion together into a rule: “If [premise] → then [conclusion].” Sometimes, a rule is not always helpful. Recognition of what the gap is might be a more useful prephrase.
The first sentence is a comparison between the pull stroke between an S-shaped rake and a straight-handle rake. The S-shaped rake will reduce compression on the spine to 1/5 of what is it with the latter.
The next sentence starts with a “however” which makes me think there is some information that is different or runs contrary to the information presented before in some way (pivoting). Reading on, we learn that on the push stroke, the s-shaped rake exerts 5x more pressure than the straight rake.
In the next sentence, we learned that the compression in the pull/push for the straight rake is not dangerous, but it is above the danger lever for the s-shaped rake. So far all of this is just information/premises. The next sentence begins with “therefore,” and states that straight rakes are better than s-shaped rakes for minimizing the risk of spinal injury. This is our conclusion.
What is the gap here? Well, jumping from compression stress injuries on the spine to all injuries on the spine. And also, the statistics of risk are equal: X being 5 times Y and Y being 1/5 of X is equal. Why should we prefer a straight-handled rake if the risk of injury for pulling in one and pushing in the other is the same? Our rule needs to relate stress caused by pushing (because that’s the one where s-shaped rakes are worse) to factors associated with all spinal injuries when raking. Not only that, but it needs to explicitly say that the two go hand in hand, or compression related to the push (and not pull) stroke is one of the most important factors in all spinal injuries caused by raking.
Correct Answer Choice (A) This answer choice is relating compression from pushing and spinal injuries in a causal relationship by saying that compression stress from pushing rakes is the only cause of spinal injuries from raking. So, of all the injuries that happen with raking, spinal injuries are only caused by compression stress from pushing the rake, which means that we can conclude that straight rakes are better for minimizing spinal injuries in general.
Answer Choice (B) This is not the correct answer. It’s not relevant to making our conclusion valid – sure, a gardener can still get spinal injuries by using even a straight-handled rake. If we plug this back into our stimulus, it doesn’t really do anything.
Answer Choice (C) This is describing what is happening with the redesign’s flaw. If this was a principle question, this would be a great answer choice. However, this is an SA question and our conclusion has nothing to do with a redesign having no gain in efficiency. We’re making a judgment claim on which rake is better as it relates to a specific category of injuries.
Answer Choice (D) This is basically saying that some strokes have to be pull strokes, which means that there is some risk of compression injury with a straight handle. This attempts to weaken our answer choice but fails because some risk while raking is compatible with our conclusion (it’s about lowering risk, not eliminating it).
Answer Choice (E) This is pointless to our argument. We’re trying to say that one is better than the other, not introduce a third option.
This is a sufficient assumption question: this question stem includes “properly drawn if which one of the following is assumed?”
Sufficient assumption questions tend to be very formal. We’re looking for a rule that would 100% validate the conclusion, specifically by bridging the premise and conclusion through the rule. Not only are we extrapolating the rule from our argument, but we’re also using that rule to render the argument “valid.” The way to prephrase our answer choice is by tying our premises and conclusion together into a rule: “If [premise] → then [conclusion].” Sometimes, rules are a little too chunky and don’t capture the gap accurately. This question is a great example of why.
The first sentence introduces a phenomenon: impact craters have been found everywhere, but more are found in geologically stable regions. Basically, in the most stable regions, for example in Canada, we find a lot more impact craters than we do in Japan. This does make sense. Being geologically stable has nothing to do with the frequency and location of meteor impacts; earthquakes, tsunamis, and tectonic movements don’t impact space activity.
Our next sentence is our conclusion, explaining this phenomenon: the level of land destruction is lower in these regions, so presumably, the impact craters would remain intact. This seems completely logical. However, this is one possible explanation. The argument hasn’t done anything to address any other explanation. Either way, we can’t say the explanation for the higher number of these impact craters being visible in these regions must be because they weren’t destroyed by geophysical processes. To bridge this gap, we have to block other explanation with a rule like: “If impact craters are visible in areas that are geologically stable, it must be because they weren’t destroyed.” Remember that our correct answer choice may not mirror our rule; it could also block any other explanation.
Answer Choice (A) This is not correct. If a meteor obliterates a trace of another meteor, how does that help validate that the craters weren’t destroyed in the more stable regions? This does not help validate our conclusion.
Answer Choice (B) The rates fluctuating don’t help support the explanation for why more craters are visible in stable areas. In fact, the rate of geophysical destruction varying within any region might actually hurt our premise.
Answer Choice (C) Just because there are more meteors striking down, that doesn’t support our conclusion for the explanation for visible meteors in stable areas. In fact, this could weaken the argument - what if all of the increased meteor showers hit stable regions?? Then the explanation the conclusion gives would be totally false.
Correct Answer Choice (D) It’s rejecting the overlooked possibility that the other explanation for why there are more craters in stable regions is simply because there were more craters that hit the areas. With this premise inserted in our argument, we can properly draw the conclusion: since craters were hit everywhere evenly and are more visible in stable regions, the explanation must be that these stable areas aren’t prone to destruction and the craters are preserved.
Answer Choice (E) If one area is studied more than others, that does not help support the explanation. This is not an issue of discovery or what craters are not known. We have to accept the premise that more craters show up in stable regions.
This is a sufficient assumption question because the question stem says: “conclusion is properly drawn… which one is assumed?”
Sufficient assumption questions tend to be very formal. We’re looking for a rule that would 100% validate the conclusion, specifically by bridging the premise and conclusion through the rule. Not only are we extrapolating the rule from our argument, but we’re also using that rule to render the argument “valid.” The way to prephrase our answer choice is by tying our premises and conclusion together into a rule: “If [premise] → then [conclusion].” Sometimes though, rules are a little too chunky and don’t capture the gap accurately. This question is a great example of why.
The first sentence is pretty straightforward: photovoltaic power plants (PVP) create electricity through sunlight. We’re also told that it’s cheaper than it was 20 years ago and included the costs of construction and operation in this statement. The passage then notes that corresponding costs have increased for traditional power plants (TP). Okay, all good so far – the author is noting the change of costs for each separately.
The conclusion is that PVP is less expensive than TP. Wait – how can we draw a comparison between the two? We only know what’s going on with their respective costs. We can’t make a comparison between the two when we only know what’s going on within the two respectively. Let’s walk through this:
Imagine in 2003 the cost for PVP was $500. Since then, it’s decreased by 1/10, so the cost today is $50.
Now, for TP, in order for our conclusion to be true, the cost of TP in 2023 would have to be above PVP’s costs in 2023. So, something like: in 2003, the cost for TP was $600, and now the cost is $6000. (A note that “increasing” is a very vague term on the LSAT – we can’t really tell by how much something increased.)
But – does that have to be the case? NO!! It’s totally possible that the cost for TP was $20 in 2003, and then increased to $45. This goes against the conclusion. The problem here is that the premises are only giving information for comparison within the group, and our conclusion is about what’s going on between the groups. There is a mismatch.
If you’re having trouble seeing the comparison issue, let’s take another context. Let’s use the LSAT score as an example. What this argument is saying is the following... I took the LSAT twice and my score went up between those two takes. My friend also took the LSAT but her score went down from her first take. Therefore, I score higher on my LSAT than my friend did.
Is this conclusion valid? Not really - it could be that I went from a 150 to a 154; but my friend went from a 160 to a 159. This is the issue: having information about what’s going on with me and separately with my friend doesn’t allow for a conclusion comparing my score with my friend’s score.
Back to our argument, to close the gap here, a rule feels a little complicated. We need something explicit to address this gap. Something that draws a comparison between the two groups, not just within them. Something like, even 20 years ago, the cost for PVP was way lower than TP plants.
Answer Choice (A) This answer choice is just a repetition of what was stated in the stimulus above. A repetition of our stimulus isn’t going to help us out here.
Answer Choice (B) This isn’t correct – we don’t care about electricity produced 20 years ago. The technology could have improved and PVP produces much more; on top of this, the conclusion is about the cost of electricity now.
Answer Choice (C) Plugging this back into our stimulus will show how this is completely useless to our argument. This answer choice is basically saying that PVP and TP technology works in different ways; so, what? It’s possible to have two ways of doing something and both of them are great. This doesn’t make our conclusion valid.
Correct Answer Choice (D) We’re saying that even back then, PVP was 10 times less than TP. If PVP costs went down by a tenth and we also know that TP costs have increased, then we can properly draw our conclusion.
Answer Choice (E) Once again, we have more information about what’s going in within the groups and no information on what these costs are relative to each other.
Here we have a five star question with a peculiar question stem. We should at least know that this is a strengthening question because it asks: Which one of the following ethical criteria, if valid, would serve to support the journalist’s conclusion. The question adds another requirement of a correct answer, stating it will support the conclusion “while placing the least constraint on the flow of reported information.” An extra qualification can throw you off, but thankfully this isn’t something you see on LR questions anymore. The LSAT writer wants this requirement to lurk in the back of your mind and distract you, but when an extra requirement is added, you should focus on approaching the question as normal and only bring in the second requirement if you have multiple answers which meet the first.
The stimulus tells us about a journalist in a civil war who found evidence that the government was responsible for refugees starving. Unfortunately, government censors removed any mention of the government from the journalist’s report. The journalist concluded (alarm bells should be going off for you here) that it was ethically permissible to file the censored report, because (more alarm bells) it would be preceded by a notice that it had gone through government censors. This is a rare case where the stimulus is more straightforward than the question stem.
The key sentence is the last one, it gives us the conclusion which a correct answer must support, and the journalist’s reasoning for it. She believes that although the report had been altered by the government, it is still ethically permissible to publish the censored version if readers are explicitly told it had been cleared by government censors. The correct answer will give us an ethical principle which support’s the journalist’s reasoning, while incorrect answers will fail to do this or even outright contradict the journalist’s conclusion. Let’s move to the answers now:
Answer Choice (A) The first thing we should do when judging this answer is ignore “it is ethical in general to report known facts”, because it is a broad statement which adds very little to justify the journalist’s decision. What we are interested in is the rest of the answer, which only tells us what is unethical. The “if” should jump at you, and if we convert this answer to a conditional we get the principle that “if the omitted facts of a report might substantially alter an impression of the subject which would be congruent with the included facts, then it is unethical.” Congruent is thrown in to try and confuse you, but essentially just means consistent, i.e non-contradictory. This answer is incorrect, it greatly weakens our journalist’s conclusion because she did omit facts that would alter a reader’s impression of the government. I think it is safe to say that if you found out a government was starving people, your impression of it would be altered.
Answer Choice (B) This answer also begins with the general principle “it is ethical to report known facts” which does nothing to help us and should be ignored. Like A the rest of the answer only tells us the conditions for being unethical. This answer weakens what we wish to support, because it is true that the journalist didn’t report facts she knew which might exonerate other factors (like the rebels and nature) by assigning responsibility for the refugees starvation to the government.
Answer Choice (C) If this were a weakening question, this would be a great answer because it necessarily excludes the journalist’s action from being ethical, seeing as she filed a report which a censor had deleted unfavorable material from. But this is a strengthening question, and therefore C is incorrect.
Correct Answer Choice (D) This answer includes an if statement within an unless statement. If we translate it, we can get “If there is a censorship warning, then it is not true that if you report censored material, it is unethical.” This supports the journalist’s conclusion by supporting her reasoning that a censorship notice negates any ethical issues with publishing censored material.
Answer Choice (E) You know what to omit here. This answer is mostly a repeat of D, but with the addition of an AND statement. To be ethical a censored report’s reported facts (i.e the ones that weren’t censored) must now also not give a misleading impression by themselves. This answer is essentially a combination of A and D, and is incorrect for the same reason that A is. The reported facts by themselves, independent from the censored facts and the censorship notice, do give a misleading impression of the government. If you just read the facts that were included, you wouldn’t get the impression that the government was starving people.
This is a weakening question, we know this because the question stem asks: Which one of the following, if true, weakens the researchers’ argument?
This stimulus has the common LSAT trope of “some people used to believe X, now they believe Y, because of Z.” Our job is to weaken Z. Before scientists assumed that all Dinosaurs were cold-blooded like modern reptiles, but now they believe some may have been warm-blooded because there are fossils in the northern arctic and it is too cold there for cold-blooded animals. The conclusion is a hypothesis meant to explain how dinosaur fossils could be found in the arctic if cold-blooded animals can’t survive the winter there. This argument has some large assumptions it makes to reach the warm-blooded hypothesis; for example, that the dinosaurs remained in the arctic all year long, and that the climate was the same then as it is now. A good answer is going to pick up on one of these assumptions and undermine it, directly weakening the relationship between the support and the conclusion. Let’s take a look at the answer choices:
Answer Choice (A) This supports the argument. Since the stimulus tells us that all of today's reptiles are cold blooded, all this answer adds is that most are confined to particularly warm climates, further strengthening the relationship between cold-blooded animals and warm climates.
Answer Choice (B) This answer brings in size, hoping to get you thinking about how size might relate to the ability to survive in cold environments. The truth is that we just aren’t given any information about how size might relate to the scientist's conclusion, and therefore are not entitled to make any inferences that might weaken the argument.
Answer Choice (C) If the stimulus claimed that Dinosaurs couldn’t have lived in the arctic because there was nothing for them to eat, then the existence of cold-resistant plants might be relevant; but it specifically states that the issue is that cold-blooded animals would freeze to death in an arctic winter. This answer is particularly bad because the plants having been cold resistant further suggests that the arctic was extremely cold in the past as well, which is an assumption that supports the scientists argument.
Correct Answer Choice (D) This answer picks up on one of the assumptions we identified in the stimulus; that the dinosaurs remained in the arctic in all seasons. The researchers’ support specifically states that it is the arctic winters that are the problem. A fossil does not tell us in what season the animal died and whether it lived all year in the area in which it died. This answer undermines this assumption that the dinosaurs lived in the arctic during the winter, from which the researcher’s draw their warm-blooded hypothesis. If the dinosaurs would need to continually migrate, we have a reason to believe they may not have remained in the arctic during the winter and hence may have been cold-blooded like all other reptiles.
Answer Choice (E) This would be a good answer if it were a strengthening question. Like D it picks up on an assumption the argument makes, in this case that the arctic climate was similar during the period of the Dinosaurs. By explicitly stating that it was in fact similar, it directly strengthens the warm-blooded hypothesis.